LSAT and Law School Admissions Forum

Get expert LSAT preparation and law school admissions advice from PowerScore Test Preparation.

 Sherry001
  • Posts: 81
  • Joined: Aug 18, 2014
|
#20193
Hello;
I had trouble choosing between A and B . could you please check my reasoning for choosing A over B?
Thanks so much!

1- some doctors believe that a certain drug reduces the duration of Vertigo as there has been a decrease since the drugs use.

2- no significant changes in the duration of vertigo since shortage.

C: The drug has no effect on the duration of vertigo.

Before looking at the answer choices, I felt the conclusion does not really follow from the evidence. To say the drug has no effect is too strong. what if the drug worked and that's why there were no significant changed?

A) negated this would destroy the argument. so therefore the correct answer
.
B) This was super tempting to me. And the only reason I did not choose this was because the author states that there were "no significant changes", meaning there were at least some changes, just not enough to prove the effectiveness of the drug. so negated this would not destroy the authors argument.
 Ricky_Hutchens
PowerScore Staff
  • PowerScore Staff
  • Posts: 59
  • Joined: Oct 12, 2015
|
#20197
Hi Sherry,

Good job! Your breakdown of the stimulus is great and it got you to the right answer.

I would like to point out that choice B is weaker than I think you realized. Notice that the author's argument in the stimulus is completely based on the idea that the 3 month shortage had no effect on the average duration of vertigo. But choice B is concerned with any change that might have occurred since the drug's introduction.

The author might have agreed with choice B, but it was in no way required for him to reach his conclusion. Therefore it's not an assumption his conclusion depended upon.

Ricky Hutchens
 kmpaez
  • Posts: 12
  • Joined: Sep 18, 2017
|
#43260
Could you please explain why answer choices C and D are incorrect? They seemed like good defender assumptions to me.
 Emily Haney-Caron
PowerScore Staff
  • PowerScore Staff
  • Posts: 577
  • Joined: Jan 12, 2012
|
#43306
Hi kmpaez,

For answer choice C: For the argument to work, we don't need 3 months to be the best length of time to use, we just need it to be an adequate length of time. Even if longer would have been better, as long as 3 months is at least adequate, the argument still holds. That's why C is incorrect, here; it isn't a required assumption because even if a longer time was better, it wouldn't kill the conclusion.

For answer choice D: Even if the opposite of D were true (I.e., changes in diet and smoking were responsible for some of the change in vertigo), would the argument still work? The answer here is yes; we don't know how diet and smoking changes might have varied during that time, we don't have any information about it, so D ends up pretty irrelevant. If you can explain your reasoning for D, we can tailor the answer more directly to your thinking.

Hope this helps!
 cmnoury1221
  • Posts: 21
  • Joined: Sep 11, 2019
|
#68591
Hello,
I am confused as to why answer choice (A) is correct here: If you negate (A) and say If a drug made a difference in the duration of vetigo, a three-month shortage of that drug would NOT have caused a significant change in the average duration fo vertigo" .. how does that destroy the argument? I would appreciate some help on how the negation technique plays out here.

Thank you!
Carolyn
 Jeremy Press
PowerScore Staff
  • PowerScore Staff
  • Posts: 1000
  • Joined: Jun 12, 2017
|
#70945
Hi Carolyn,

You're got the right negation of answer choice A, but determining the impact of that particular negation on the conclusion is a little harder than it looks on the surface. Answer choice A is really the contrapositive of my prephrase here (remember, contrapositives are logical equivalents). So let's first consider what the contrapositive of answer choice A is: "If a three-month shortage of the drug did not cause a significant change in the average duration of vertigo, then the drug did not make a difference in (i.e. did not have an effect on) the duration of vertigo." The negation, or logical opposite, of that conditional is: "Even if a three-month shortage of the drug did not cause a significant change in the average duration of vertigo, the drug might still have had an effect on the duration of vertigo." That logical opposite severs the link between the premise and the conclusion in the argument, thus invalidating the conclusion. The logical opposite is telling you that the drug might still have had an impact on duration despite a three-month shortage not causing a significant change in vertigo (for example, maybe because enough of the drug was still left in people's systems to be reducing their vertigo). Does that make it more clear? Hopefully so!

Jeremy
 lauren81390
  • Posts: 3
  • Joined: Jan 21, 2022
|
#98062
I'm still confused as to why A is the correct answer - I'm not understanding how the negation technique helps make this the correct answer.
User avatar
 Paul Popa
PowerScore Staff
  • PowerScore Staff
  • Posts: 64
  • Joined: Sep 20, 2022
|
#98082
Hi Lauren,

I'd be happy to help with that! Overall, the author is giving us just one piece of evidence to support their conclusion: during a three-month shortage, there was no significant change in the average duration of the vertigo. From this, the author concludes that the drug has no effect. But as you may have noticed, this argument is not airtight. For example, maybe the drug has a long half-life and is able to maintain its effects for at least a year after the last dosage. We realize, then, that the author believes this piece of evidence is sufficient to support the conclusion.

Let's now look at answer choice A. A says that if the drug actually made a difference, we would have seen a reduction in its positive effects during a three-month shortage. Assuming this, the shortage does in fact support the idea that the drug has no effect.

When trying to logically negate an answer choice, remember that the magic word is "not": you're either taking not out or putting not in. Negated, A reads: If a drug made a difference in the duration of vertigo, a three-month shortage of that drug would NOT have caused a significant change in the average duration of vertigo." This totally wrecks the author's argument because it shows that the three-month shortage is not a sufficient amount of time to declare the drug ineffective. This is what the author is basing their entire argument on, which makes A necessary and the correct answer. Hope this helps!

Get the most out of your LSAT Prep Plus subscription.

Analyze and track your performance with our Testing and Analytics Package.